4
$\begingroup$

We have $X \sim \mathrm{Unif}[0,2]$ and $Y \sim \mathrm{Unif}[3,4]$. The random variables $X,Y$ are independent. We define a random variable $Z = X + Y$ and want to find the PDF of $Z$ using convolution. Here is my work so far:

The definition of convolution is:

  • $f_Z(z) = \int_{-\infty}^{\infty}f_X(x)f_Y(z-x)\mathrm{d} x$

We know the PDF's of $X$ and $Y$ because they are just uniform distributions. The hard part for me is finding the limits of integration. We have to solve for the constraints.

The integrand is nonzero when $3 \leq z-x \leq 4$ and when $0 \leq x \leq 2$. Together these constraints imply that $\max \{0, z-4\} \leq x \leq \min \{2, z-3 \}$.

These constraints imply that there are three cases:

  • Case 1 - $z \leq 4 \implies f_Z(z) = \int_0^{z-3}$
  • Case 2 - $4 \leq z \leq 5 \implies f_Z(z) = \int_{z-4}^{z-3}$
  • Case 3 - $z \geq 5 \implies f_Z(z) = \int_{z-4}^{2}$

My question is how to find the bounds of $Z$ i.e. what are the possible values of $Z$? Does $Z$ run from $0 \to 6$ since it is the sum of $X+Y$ and this sum will have some value for every value $\in [0,6]$?

$\endgroup$
7
  • 5
    $\begingroup$"The hard part for me is finding the limits of integration" Then write down the densities correctly, that is, including the indicator functions. Here, $$f_X(x)=\tfrac12\mathbf 1_{0<x<2}\qquad f_Y(y)=\mathbf 1_{3<y<4}$$ hence $$f_Z(z)=\int f_X(x)f_Y(z-x)dx=\int\tfrac12\mathbf 1_{0<x<2}\mathbf 1_{3<z-x<4}dx=\tfrac12\int\mathbf 1_{0<x<2}\mathbf 1_{z-4<x<z-3}dx,$$ that is, for every $z$, $$f_Z(z)=\tfrac12\int_\mathbb R\mathbf 1_{\max(0,z-4)<x<\min(2,z-3)}dx=\tfrac12(\min(2,z-3)-\max(0,z-4))^+,$$ and, only now, turn to cases to identify the RHS. This way, you cannot fail.$\endgroup$
    – Did
    CommentedOct 19, 2015 at 11:10
  • $\begingroup$@AlexanderCska "how do you evaluate min(2,z−3) and max(0,z−4)" ?? Well, by considering three cases, depending on the signs of z-5 and z-4 (what else?).$\endgroup$
    – Did
    CommentedJun 30, 2017 at 13:11
  • $\begingroup$@Did yes but $z$ is limited by bound, that depend on $x$. $3+x\le z \le 4+x$. Are these cases obtained by setting the limits for $x$, $0 \le x \le 1$. Say $ 3 \le z \le 4$ and $4 \le z \le 5$$\endgroup$CommentedJun 30, 2017 at 13:26
  • $\begingroup$@AlexanderCska Sorry but I do not get your point. (Additionally, I am not quite sure you really read my previous comment.) Maybe ask a new question, centered around the specific problem you are facing?$\endgroup$
    – Did
    CommentedJun 30, 2017 at 13:28
  • $\begingroup$@Did I am just confused how exactly do you get the limits of $z$, when looking for $min(2,z-3)$. You need to know the possible values of $z$ to get the minimum.$\endgroup$CommentedJun 30, 2017 at 13:32

1 Answer 1

4
$\begingroup$

Instead of trying to find appropriate $z$ values at the very beginning, note that $f_X(x)$ is zero unless $0\le x\le2$. Therefore $$\int_{-\infty}^{\infty}f_X(x)f_Y(z-x)\mathrm{d} x =\int_0^2f_X(x)f_Y(z-x)\mathrm{d} x =\frac12\int_0^2 f_Y(z-x)\mathrm{d} x\ .$$ You can now substitute $t=z-x$ to get $$\int_{z-2}^z f_Y(t)\mathrm{d} t$$ - see if you can take it from here.

$\endgroup$
3
  • $\begingroup$Yes, $f_X(x)$ is nonzero only when $0 \leq x \leq 2$. To find the bounds of $z$, we see that $ 3 \leq z-x \leq 4$. But, the minimum $x$ can be is $0$ and when $x=0, z = 3$. The maximum $x$ can be is $2$ and when $x = 2$, the maximum $z$ can be is $6$. Does this all imply $3 \leq z \leq 6$?$\endgroup$CommentedOct 18, 2015 at 22:56
  • $\begingroup$Yes, to get a non-zero result from the last integral we must have $z-2\le4$ and $z\ge3$. (Sorry for the delay, I was away from my desk for an hour.)$\endgroup$
    – David
    CommentedOct 19, 2015 at 0:04
  • $\begingroup$Thanks David. You're awesome$\endgroup$CommentedOct 19, 2015 at 0:05

You must log in to answer this question.

Start asking to get answers

Find the answer to your question by asking.

Ask question

Explore related questions

See similar questions with these tags.